Practice Test: Middle School Humanities (76)

Answer Key, Sample Responses, Evaluation Chart, and Score Calculation Tool

Answer Key

Fill in your answers below and then print this answer key to save your work. Alternatively, you can print the answer key first to fill it out offline as you take the practice test. Note that the correct responses will be displayed on the printed answer key, so you may want to cover them until you have completed the practice test and are ready to check your answers.

When you have finished the practice test, click on "show answers" to see how well you did on each objective. In addition, use the Evaluation Chart to determine how many questions within each objective you answered correctly.

You will not receive a score for the practice test, and there is no passing score for the practice test. However, to get a sense of how well you did, use the Score Calculation Tool to better gauge your performance and degree of readiness to take an MTEL test at an operational administration.

NOTE: When you take the actual test, you will receive a score report that provides subarea-level performance, not objective-level performance. Information about test results can be found at Score Report Explanation.

Question Number Your Response Correct Response
Related Objectives and Rationale
1 B Objective 001

The sonnet is a fourteen-line poem with a variable rhyme scheme (Correct Response B). Response A is incorrect because an ode is a poem in varied stanzas that celebrates a thing, place, or person. Response C is incorrect because a ballad is a narrative poem that commonly consists of rhymed quatrains. Response D is incorrect because free verse uses nonrhyming, unmetered lines.

2 D Objective 001

The phrase "cultured hell" juxtaposes a positive descriptor ("cultured") with a negative descriptor ("hell") (Correct Response D). Response A is incorrect because "cultured hell" is not an example of personification, nor does the speaker express guilt. Response B is incorrect because the word "culture" implies neither decadence nor wastefulness. Response C is incorrect because an ironic tone would suggest a difference between the words used and their intended meaning.

3 C Objective 001

Personification is the endowing of a nonhuman entity with human attributes, and here Glück personifies a wild iris: "Whatever / returns from oblivion returns / to find a voice," Glück writes in the sixth stanza, personifying the iris's experience of rebirth and attitude of hope (Correct Response C). Response A is incorrect because Glück focuses solely on the wild iris's relationship to its own life cycle, not that of humanity. Response B is incorrect because Glück does not include references to the cycles of perennials. Response D is incorrect because the flower is the speaker in the poem.

4 C Objective 001

Climax is the dramatic apex of plot, the moment when the protagonist is forced into irreversible action—such as Rainsford leaping into the sea (Correct Response C). Response A is incorrect because the denouement is the conclusion of the plot, when the protagonist's crisis has been resolved. Response B is incorrect because the exposition sets the stage for the conflict; in the excerpt, the conflict is fully underway. Response D is incorrect because the resolution is when the conflict is settled.

5 C Objective 001

Alexander explains the crossover, first providing a definition and two examples, to build up to the speaker's personal connection to the basketball move (Correct Response C). Response A is incorrect because, outside of the first stanza, Alexander uses a decidedly subjective tone, deeming "Deron Williams's crossover dot dot dot nice" and "Allen Iverson's dot dot dot deadly." Response B is incorrect because the excerpt never discusses the importance of exercise. Response D is incorrect because an extended metaphor uses one object to represent something else; in the excerpt, the crossover signifies the crossover.

6 B Objective 001

Contrasting imagery of the unencumbered, idyllic natural world with imagery of the violent cage, Dunbar develops the theme of yearning for freedom and equity (Correct Response B). This is evident in the lines "For he must fly back to his perch and cling / When he fain would be on the bough a-swing." Response A is incorrect because the poem does not describe the relationship between the speaker and the caged bird; instead, the caged bird is an extended metaphor for the speaker. Response C is incorrect because while the poem references the bird's singing, the main focus is the cause of the singing rather than the emotion it expresses. Response D is incorrect because, while Dunbar does evoke the beauty of the natural world in the first stanza, the second stanza develops the idea that it is out of reach for the "caged bird" and, by extension, the speaker.

7 B Objective 001

Eliot's diction, with words like "flattering" and "egoism," reveals the import of the parable: that viewing events through one's own perspective can lead to biased, self-important interpretation (Correct Response B). Response A is incorrect because Eliot's narrator regards the different perspectives on events as resulting from "flattering illusions" rather than unique viewpoints. Response C is incorrect because it omits the significance of the viewer's role in perceiving and interpreting events. Response D is incorrect because the excerpt does not discuss the significance of the events.

8 A Objective 001

Tropes are recurrent motifs, and in fairy tales and folktales significant numbers are some of the most recognizable; here, the author remarks on this trope by repeating the word "seventh" or "seven" seven times in the span of nine lines (Correct Response A). Response B is incorrect because fairy tales often contain generalized or abstract scenery, such as the forest or the castle (here "the wildwood"). Response C is incorrect because the journey is not a trope but an element of plot. Response D is incorrect because the author makes no mention of wicked stepparents.

9 D Objective 001

Stream of consciousness is a type of narration that attempts to represent a character's mental process realistically through an associative and uninterrupted flow of sensory details, emotions, memories, and thoughts. Correct Response D blends sensory associations ("Blue dusk, nightfall, deep blue night") and seemingly random thoughts ("Who to clear it?" or "Call: no answer") in a way characteristic of stream of consciousness. Response A is incorrect because it is an example of first-person narration, in which the character is describing an event. Responses B and C are also examples of first-person narration in which the characters engage in introspection, but their thinking proceeds in a linear, organized fashion, uncharacteristic of stream of consciousness.

10 TBD Objective 002

Item 10 coming soon.

11 TBD Objective 002

Item 11 coming soon.

12 C Objective 002

Response C is correct because the excerpt describes how the physical properties of some birds' feathers result in the appearance of a blue color. Response A is incorrect because the excerpt does not define or classify different types or parts of feathers. Response B is incorrect because while the excerpt does discuss how scientists found an answer to a question, this serves to describe the causes of a phenomenon rather than present a solution to a problem. Response D is incorrect because the excerpt focuses on describing one type of feather rather than discussing similarities among or differences between feathers.

13 D Objective 002

The author begins the excerpt by noting that scientists were trying to "find the origin of avian blue," establishing that the focus of the piece is to explain the reason bird feathers appear blue, and Prum and colleagues' study of feathers quickly brings feather structure into focus (Correct Response D). Response A is incorrect because the excerpt only briefly mentions one method scientists used to study bird feathers and then shifts to describing the structure of feathers. Response B is incorrect because the excerpt only considers one type of bird feathers. Response C is incorrect because the excerpt offers no insight into how or why some birds developed blue feathers.

14 B Objective 002

The phrases "constantly shifting ground" and "blown away in the gust" underscore the evolving state of nutrition and food science (Correct Response B). Responses A and C are incorrect because, in the context of the excerpt, these phrases are used to describe nutrition science and the food industry, not the American public. Response D is incorrect because the excerpt does not describe scientific processes and expresses skepticism about the idea that nutrition science is becoming more reliable.

15 D Objective 002

Inferences combine textual evidence and the reader's own reasoning to come to new conclusions. Pollan's use of phrases like "'latest science,'" in those veracity-challenging quotation marks, combined with the content of the excerpt suggest the piece is skeptical about scientists' claims about health and diet (Correct Response D). Response A is incorrect because the excerpt is critical of the diet industry's obsession with new health fads. Response B is incorrect because, while Pollan notes that the food industry "amplif[ i e s ] the 'latest science'" to market products, he does not suggest that marketing is the primary motivation for scientific research. Response C is incorrect because Pollan cites examples—like the changing status of hydrogenated oil—to challenge the idea that the latest research is necessarily correct or beneficial.

16 C Objective 002

The excerpt defines apprenticeship and explains how one works; its overarching purpose is to educate readers (Correct Response C). Response A is incorrect because, though the text describes apprenticeships of varying lengths, that is a small detail and not the text's main point. Response B is incorrect because the text is written in objective language, without any biased or subjective points; there is no call to action or pitch to the reader. Response D is incorrect because the text does not mention any careers that require apprenticeships.

17 A Objective 002

A summary is a brief, objective overview of a text. When a summary is used within a report, it should efficiently and accurately survey the text (Correct Response A). Response B is incorrect because it focuses on a minute detail: the smelting plant where Sandy's father worked. Response C is incorrect because it omits important details such as the location of the Bungalow as well as the year of Sandy's birth. Response D is incorrect because it contains editorializing details ("Hugh was… a charming host" and "Janet threw lovely tea parties").

18 B Objective 002

In the excerpt, the word "lineaments" refers to pronounced facial characteristics (Correct Response B). Responses A, C, and D are incorrect because these words do not denote significant features.

19 A Objective 002

The author's point of view is evident in the tone in which the author writes about the subject. By paying attention to elements of style, such as diction and syntax, a reader can determine the author's point of view. Words like "salamandery" and phrases like "they too migrate" capture the author's awe and admiration of the tiger salamanders, for both their cleverness and their hardiness (Correct Response A). Response B is incorrect because the author repeatedly cites the tiger salamanders' competency. Responses C and D are incorrect because the author is neither matter-of-fact nor somber with his use of the word "salamandery" and his description of the "ephemeral" breeding pond.

20 C Objective 003

When English language learners are literate in their home language, they learn English more easily (Correct Response C). Responses A, B, and D are incorrect because the research does not support these statements.

21 A Objective 003

Common among bilingual learners, language transfer involves applying the linguistic features of one language to another. In some languages, for example, the noun precedes the adjective that modifies it (Correct Response A). Response B is incorrect because the input hypothesis refers to language acquisition: language learners progress when they can understand language that is more advanced than their current level of development. Response C is incorrect because code meshing refers to the blending of one aspect of one's language with another aspect of the same language. Response D is incorrect because formulaic language refers to utterances and pause fillers, expletives, idioms, and other types of form-fixed speech.

22 B Objective 003

Idiomatic expressions are phrases or sayings that are often particular to speakers from a specific culture or historical period. Because of their cultural specificity and frequent use of figurative language, idiomatic expressions can present challenges for English learners. To best support comprehension of an idiomatic expression encountered in a short story, a student should be taught the meaning of the idiomatic expression, as well as how it is used in various contexts (Correct Response B). Response A is incorrect because the meaning of an idiomatic expression is often figurative and dependent on cultural context rather than the literal meaning of the individual words. Response C is incorrect because drawing an idiom would not necessarily help clarify its meaning in the context of the short story. Response D is incorrect because providing a minilesson on the difference between literal and figurative language would not provide students with the cultural context necessary for understanding the idiomatic expression.

23 B Objective 003

Inferences are made when a student's baseline knowledge is applied to their interpretation of a text. By parsing the text and considering what the author may be implying rather than explicitly stating, students use evidence and their own reasoning to come to new conclusions (Correct Response B). Response A is incorrect because the sentence stems focus on the students' thoughts rather than the author's purpose. Response C is incorrect because neither sentence stem prompts the students to consider the central idea of the text. Response D is incorrect because prompting students to reflect on their thoughts on a topic prior to reading would not support their analysis of particular word choices.

24 A Objective 003

The prefix "con-" means "together" or "with." Knowledge of this prefix would aid in the reader's ability to determine that "confluence" means "the junction of two rivers" or "a process of merging" (Correct Response A). Responses B, C, and D are incorrect because these words do not denote joining or combining, so knowledge of the prefix would be less helpful.

25 D Objective 003

Pre-teaching vocabulary words before students encounter them in a lesson or text aids in comprehension, so it is especially impactful for introducing difficult, conceptual vocabulary (Correct Response D). Responses A and B are incorrect because high-frequency words will be used throughout a text and thus, regardless of their phonetic regularity or homonymity, students will grasp them more readily. Response C is incorrect because the length of a word does not inherently make it valuable for pre-teaching.

26 A Objective 003

Anticipation guides are designed to activate background knowledge and promote students' thinking about a topic (Correct Response A). Responses B and D are incorrect because anticipation guides focus on broad subjects or ideas, not individual details or organizational structure of a specific text. Response C is incorrect because anticipation guides help students think about what they already know about a topic, not whether the author is a reliable source.

27 C Objective 003

Understanding base words and prefixes enables a reader to define unfamiliar words. Knowing, for instance, that the prefix "dis" expresses negation and the word "cord" comes from the Greek for "string of a musical instrument" would help a reader understand that "discordant" means "inharmonious or jarring" (Correct Response C). Response A is incorrect because onsets—the consonants before the vowel—and rimes—the letters after the onset—do not supply connotative information. Response B is incorrect because connotative meaning refers to the valence or tone of a particular word, not its definition; it would be unlikely that a reader would know a connotative meaning without first knowing the denotative meaning. Response D is incorrect because letter-sound correspondences help readers pronounce words, not identify their meanings.

28 D Objective 003

"In hot water" is an idiomatic expression that indicates being "in trouble." Parsing an idiomatic expression requires evaluating the context clues in the sentence, especially since many idiomatic expressions are borne of metaphors (Correct Response D). Response A is incorrect because idioms about water or heat do not have common definitions; for instance, "things are heating up" may indicate increasing interest in a romantic relationship and "it's water under the bridge" communicates a "let bygones be bygones" attitude. Response B is incorrect because understanding the relationship between the words in the phrase "in hot water" would not help the student determine the meaning of the idiom. Response C is incorrect because the three words in the phrase do not have connotative meanings.

29 A Objective 003

The sentence structure ("Though they were once…") suggests a change in the nature of pay phones. The word "rare" would provide the best clue for the meaning of "ubiquitous," a word that denotes omnipresence (Correct Response A). Response B is incorrect because "expensive" is not in opposition with "ubiquitous," nor does it provide any context clues; "expensive" means costly. Response C is incorrect because "found" creates a nonsensical and inaccurate sentence, one that ignores the temporal determinant "Though they were once." Response D, while perhaps accurate, does not provide a clue for the meaning of the word "ubiquitous" since something can be both ubiquitous and unnecessary.

30 C Objective 003

Dyslexia is a learning disability characterized by difficulties with accurate and fluent word recognition. Individuals with dyslexia struggle with spelling and decoding (Correct Response C). Response A is incorrect because using a graphic organizer to track main ideas in a text would not address the word-level challenges of students with dyslexia. Response B is incorrect because using context clues to determine the meaning of words would not address the students' difficulties with accurate and fluent word recognition. Response D is incorrect because students with dyslexia may have strong oral language skills, and focusing on comprehension when reading a text aloud would not address the students' reading challenges.

31 B Objective 003

To support students from diverse backgrounds, instructional approaches must be intentional about the incorporation of multicultural examples and texts (Correct Response B). Response A is incorrect because modeling the skill does not incorporate diverse perspectives. Response C is incorrect because a graphic organizer only focuses on the foreshadowing students identify in the teacher-selected story, which may not address students from diverse backgrounds. Response D is incorrect because assigning students to write their own narratives does not guarantee that the content will be culturally relevant to the students.

32 C Objective 004

Usage errors may occur when homophones—words that sound alike but have different meanings—are incorrectly used. "Principle" and "principal" are commonly confused homophones. A "principal" is the head of a school or another organizational lead whereas a "principle" is an upheld value, moral, or ethic (Correct Response C). Responses A, B, and D are incorrect because they do not contain errors in usage.

33 D Objective 004

Sentence 1 is a compound sentence that is incorrectly punctuated. To correct the error, the independent clause "Penguins are most often associated with Antarctica" requires a semicolon after "Antarctica"; additionally, a comma is required after the introductory element "however" (Correct Response D). Responses A and B are incorrect because a comma is required after the introductory element "however." Response C is incorrect because a semicolon or period is required after an independent clause in the absence of a coordinating conjunction.

34 D Objective 004

Reverse outlining consists of reviewing a drafted essay for its organization; each paragraph of the essay is considered for its topic and its role in advancing the argument (Correct Response D). Response A is incorrect because reverse outlining does not entail reviewing transitions. Response B is incorrect because reverse outlining focuses on the purpose and development of each paragraph in an effort to understand how ideas are developed throughout the essay, not simply in its concluding statement. Response C is incorrect because a reverse outline primarily helps a writer understand if their ideas are advancing in an appropriate order, not whether their supporting evidence is effective.

35 C Objective 004

To examine how a transformation or evolution has occurred, a cause and effect organizational structure, which focuses on exploring causal relationships, is best (Correct Response C). Response A is incorrect because the student is not comparing two entities. Response B is incorrect because the student is not identifying a problem that requires a proposed solution. Response D is incorrect because the student is interested in how streaming services have changed the music industry, not necessarily when.

36 B Objective 004

Proper nouns, first person pronouns, and the first letter of the first word of a sentence require capitalization. "Chocolate Milk," which appears in the middle of a sentence, is not a proper noun, so its capitalization is erroneous (Correct Response B). Response A is incorrect because the first letter in the first word of a sentence and the proper nouns ("Bentonville, Arkansas" and "Frank Lloyd Wright") are appropriately capitalized. Response C is incorrect because the first letter in the first word of a sentence and the proper noun ("The Beatles") are appropriately capitalized. Response D is incorrect because the first letter in the first word of a sentence and the proper noun ("Susans") are appropriately capitalized.

37 A Objective 004

Response A is correct because working with classmates to create a class Web page with school news articles provides students with an opportunity to plan, draft, revise, and publish their writing for a specific purpose and audience of peers. Response B is incorrect because delivering a class presentation is not an authentic writing task, nor does it make the most effective use of digital technology. Response C is incorrect because there is not a specific audience for students' commercials, and this project would not give students an opportunity to share their writing. Response D is incorrect because participating in a discussion on an online message board would not engage students in the process of planning, drafting, and revising their writing for a specific purpose and audience.

38 B Objective 004

The first sentence of a paragraph in an informative essay functions as the topic sentence, stating the central idea that will be developed in the paragraph; subsequent sentences should advance that topic. Here, the topic sentence points to both coffee's global ubiquity and its history (Correct Response B). Response A is incorrect because caffeine's role in coffee's popularity does not develop the central idea. Response C is incorrect because the sentence preceding the blank discusses coffee's popularity in the eighteenth century, and backtracking to discuss the seventeenth century would not support the logical development of the central idea. Response D is incorrect because the sentence that follows the blank does not develop the notion that coffee must be grown in tropical climates.

39 C Objective 005

An effective research question is neither overly broad nor overly narrow; it must be open enough to require the span of an essay to answer it (Correct Response C). Responses A and B are incorrect because they do not invite elaboration or development. Response D is incorrect because it is too broad—there are too many sources of renewable energy and nonrenewable energy to research for a typical assignment.

40 D Objective 005

A university faculty Web page for Yasmina Reza is a reliable source of information about the playwright (Correct Response D). Response A is incorrect because an online retail listing of a play by Reza would not provide information about her. Response B is incorrect because quotations by Reza could be taken out of context and do not necessarily pertain to her biography. Response C is incorrect because Wikipedia is an editable, collaborative source and is not appropriate for citing in an academic context.

41 D Objective 005

Known as Boolean operators, words like "and" and "or" and "not" help search engines coordinate the relationships between search terms. To locate information about whether or not magnolia trees are native to Massachusetts, the student would need to use "AND" between the terms "Massachusetts," "native plant," and "magnolia tree," which would include all three terms in the search (Correct Response D). Response A is incorrect because including "OR" between search terms will yield results too numerous and potentially too broad. Response B is incorrect because "magnolia tree native" is not a useful keyword, and separating "Massachusetts" and "state" with "OR" will lead to ambiguous results that do not necessarily focus on Massachusetts. Response C is incorrect because "what state" is an ineffective keyword given magnolia trees are native to many states.

42 A Objective 005

As a primary source, a collection of letters written by soldiers would contain numerous subjective accounts of the experience of living and fighting during the American Revolution (Correct Response A). Response B is incorrect because an encyclopedia entry, a secondary source, would provide an overview of the period, rather than focusing on the Revolution's effects on society. Response C is incorrect because a military history book's focus would not include societal effects. Response D is incorrect because a scholarly article on the influence of the Enlightenment on the American Revolution would not give the student the most relevant information on the Revolution's effects on society.

43 A Objective 005

Effectively integrating material from a source using a quote requires appropriate attribution, as well as a signal phrase introducing the quoted text (Correct Response A). Response B is incorrect because the sentence does not introduce the source of the quote, and the lack of transition into the quote after the previous sentence is jarring. Response C is incorrect because, though the sentence smoothly incorporates the quoted text into the paragraph, it does not provide an attribution for the quote. Response D is incorrect because, though it effectively provides an attribution for the quote, it uses a colon incorrectly when introducing the quote.

44 B Objective 005

The Modern Language Association style guidelines specify that date of access is required for Works Cited page citations of electronic periodicals (Correct Response B). Response A is incorrect because the publication date is already included in the citation. Response C is incorrect because the name of the publisher is not required for this type of citation. Response D is incorrect because the author's name is already included in the citation.

45 D Objective 006

With any public speaking engagement, rehearsing the speech and ensuring an appropriate speed of delivery is important for ensuring audience comprehension and speaker confidence (Correct Response D). Response A is incorrect because not all oral reports require the incorporation of visual aids and props. Response B is incorrect because using a thesaurus may make the student uncomfortable or lead to misuse of unfamiliar words as the student attempts to incorporate new vocabulary into their report. Response C is incorrect because memorization does not lead to ease while speaking and may result in stiff delivery style.

46 B Objective 006

To support a presentation, digital media must be relevant to the topic. An animated graphic showcasing features of nutrition labels is a highly applicable visual that is germane to the student's topic (Correct Response B). Response A is incorrect because the student's presentation is about making healthy eating choices, not about cooking. Response C is incorrect because a bar graph comparing the amount of sugar per serving in soft drinks does not necessarily support healthy choices. Response D is incorrect because a professional athlete's diet is not necessarily relevant to the healthy eating choices of students in the class.

47 B Objective 006

Active listening entails restating and reflecting on what has been communicated, asking open-ended questions that build on that communication. By synthesizing what Workers 2 and 3 have said about pita and the naan burrito, Worker 1 shows that they have been actively listening and want to build on their coworkers' ideas (Correct Response B). Responses A and C are incorrect because, though they relate to the topic of the conversation, they are personal non sequiturs that do not build on their coworkers' ideas. Response D is incorrect because it forecloses new ideas; based on the coworkers' conversation, pita seems a non-negotiable component of the menu.

48 C Objective 006

When presenting, a speaker must consider their audience and their purpose in conveying information. To support the creation of more student organizations, the school board member must be persuasive and organized to maintain fellow board members' interest. By summarizing key points at the presentation's conclusion, the speaker reiterates the aim of their presentation in a manner appropriate to a business meeting (Correct Response C). Response A is incorrect because visuals showing a budget surplus may have no bearing on the tenability of student extracurriculars. Response B is incorrect because a student council president's endorsement would not be meaningful at a school board meeting. Response D is incorrect because pausing to ask the board members to reflect on personal experiences does not provide relevant information about the new policy.

49 D Objective 006

Group projects require collaboration and cooperation, with each member completing a portion of the work and disseminating their findings to the group. Building off group members' comments during a discussion would be an ideal way for the student to present information on Honduran cuisine (Correct Response D). Response A is incorrect because it does not entail the student sharing what they've learned about Honduran cuisine. Response B is incorrect because reading from prepared lecture notes does not promote discussion and collaboration between group members. Response C is incorrect because providing a list of sources does not give the groupmates access to the student's research findings about Honduran cuisine.

50 A Objective 006

To determine whether the speaker's message is based on fact or opinion, the student must carefully evaluate the veracity of the speaker's claim by considering the types of supporting details provided: Are those details objective and based in concrete evidence or subjective and rooted in opinion? (Correct Response A). Response B is incorrect because vocal modulation and nonverbal clues do not reveal whether a message is based in fact. Response C is incorrect because social media presence is an extension of marketing and branding, a public relations surface used to attract viewers and followers, regardless of the speaker's commitment to facticity. Response D is incorrect because, while the speaker's credentials may be sound, those credentials do not determine whether the speaker's message will be based in fact or opinion.

51 B Objective 007

Response B is correct because the excerpt presents evidence of the celebration of public service with the observation that many people deliver speeches for recently deceased "illustrious men dot dot dot  concerning the virtues of the deceased and the successful exploits performed by him in his lifetime." Response A is incorrect because there is no evidence of social conflict or competitiveness among Roman families in the excerpt. Response C is incorrect because while the excerpt focuses on the political aspects of death in ancient Rome, it is not guaranteed that there aren't other sources that focus on spiritual traditions in ancient Rome. Response D is incorrect because the excerpt does not provide evidence that the accomplishments of Roman elites were exaggerated.

52 B Objective 012

Response B is correct because considering the author's Greek identity would allow a historian to properly analyze the anthropological tone of the excerpt. Response A is incorrect because the excerpt is based upon firsthand experiences of the author. Response C is incorrect because the author's Greek identity does not preclude him from making correct inferences. Response D is incorrect because there is no evidence in the excerpt to suggest the author's Greek identity biased their account.

53 B Objective 007

Response B is correct because King Ezana of Axum converted to Christianity in the fourth century  C E  and promoted the expansion of that religion within the kingdom. At the height of the kingdom's territorial extent in the sixth century, Axum included the northern portions of what is now the nation of Ethiopia, and Axumites sometimes used the self-designation "Ethiopian" throughout the period. Response A is incorrect because the trans-Saharan gold trade was centered in northwestern Africa, thousands of miles away from Axum. Response C is incorrect because the Kingdom of Axum was not highly involved in the Indian Ocean spice trade. Response D is incorrect because Islam expanded into Egypt in the seventh century  C E  directly from the Arabian peninsula, without yet having a presence in Axumite lands to the south of Egypt.

54 D Objective 007

Response D is correct because the Gupta Empire, which ruled much of the Indian subcontinent through the majority of the period, promoted highly significant cultural developments, including the canonization of the Mahabharata and the Ramayana as the two great epics of Hinduism, and scientific achievements such as the introduction of the base 10 numeral system. Response A is incorrect because the military achievements of the Gupta rulers made possible the unification of most of the subcontinent but did not extend to Southeast Asia. Furthermore, military power does not usually represent the type of distinction that historians mean when they designate a "Golden Age." Response B is incorrect because the achievements of the Gupta Empire in these centuries were not based on export-driven wealth. Response C is incorrect because the Rig Veda was codified several centuries before the Gupta Golden Age of India.

55 A Objective 007

Response A is correct because Chavin artifacts reveal a polytheistic religion in many parts of the Andes mountains, marked by cities of spiritual pilgrimage (Chavin de Huantar) and sacrificial rituals that historians believe nearly 1500 people attended together. Response B is incorrect because, although Chavin architecture reveals knowledge of artistic construction methods, this civilization was not known for any extensive roads or bridges. Response C is incorrect because local chiefs and religious shamans dominated governance of the Chavin, not a strong centralized authority. Response D is incorrect because Chavin trade, though extensive throughout the Andes, did not reach many Central American societies.

56 C Objective 007

Response C is correct because the Toltec rose to power after the fall of Teotihuacan, adopting the east-west orientation and stepped pyramid style architecture in their ceremonial buildings (Pyramids B and C in Tula). Response A is incorrect because, although the Pyramid of the Sun, pictured here, is similar in the overall pyramid shape to Olmec ceremonial structures like La Venta, the Maya rose to prominence after the Olmec. Response B is incorrect because the Nazca were not known for any pyramid-type structures; rather, they are noted for their geoglyphs and less extensive ceremonial mounds. Response D is incorrect because most Incan architecture was rectangular in shape, often didn't exceed one story in height, and flowed organically into the natural surroundings.

57 D Objective 007

Response D is correct because political elites of the Nara period in Japan promoted and expanded Buddhism, while the imperial academy for future Japanese government officials became entwined with Confucian teachings and texts. Response A is incorrect because, although the Tang dynasty conquered many nearby Asian states, Japan was not included in this expansionism. Response B is incorrect because, although there was significant migration of nomadic people from China's frontier to Japan in the thirteenth century, there is no link between this population influencing Japanese religious and civil life in the eighth century. Response C is incorrect because there is no record of colonial settlement of the Chinese in Japanese territory in the eighth century  C E .

58 C Objective 007

Response C is correct because this excerpt refers to prophets and influential leaders of both Judaism (Abraham, Moses, Isaac, Jacob) and Christianity (Jesus) as believers of the same God worshipped in Islam. Response A is incorrect because there is no anti-polytheistic rhetoric in this excerpt, although Islam centers around the belief in one God. Response B is incorrect because, while the religious faiths are connected, the excerpt does not provide evidence of cultural diffusion. Response D is incorrect because, although only influential religious men are listed in this excerpt, the passage does not distinguish Muslim men and women as having different religious roles.

59 D Objective 007

Response D is correct because many of the most powerful nations at this time had established, large, and expensive standing armies that required constant funds to maintain. Wealth produced by exploitation of both humans and natural resources was integral in supporting the growth and expansion of these military forces. Response A is incorrect because, although raw materials (rubber, gemstones, opium) were acquired by European colonial powers and then manufactured outside of their native countries, this was not the main motivation in their exchange and exploitation until the  18 hundreds  during the second wave of colonialism. Response B is incorrect because profits from slavery did not precede European colonization efforts. Response C is incorrect because, although many goods produced by European powers were derived from raw materials acquired in an exploitative way and provided huge profits, these exports were not the ultimate motivation of these colonial structures.

60 D Objective 008

Response D is correct because Rhode Island was founded by Roger Williams, the writer of this excerpt and strong proponent of separation between Church and State and religious freedom, after he faced religious persecution by Puritans of the Massachusetts Bay Colony. Response A is incorrect because the only accepted religion in early colonial Virginia was Anglicanism, which was reinforced by the interconnectedness between colonial clergy and government leaders. Response B is incorrect because Georgia's colonial charter outlawed Catholicism, which both limited religious freedom and legally bound government and religious institutions. Response C is incorrect because, although the colony of New Jersey offered religious freedom similar to Rhode Island, New Jersey was not founded in the context of religious persecution.

61 B Objective 011

Response B is correct because the excerpt refers to the state not enforcing a "uniformity of religion," which is most closely related to the free exercise clause of the U.S. Constitution prohibiting Congress from making any laws that interfere with most religious practices. Response A is incorrect because the excerpt does not refer to due process of law. Response C is incorrect because the equal protection clause was created to prohibit state discrimination based on race. Most religious freedom arguments rely on First Amendment protections as opposed to Fourteenth Amendment protections. Response D is incorrect because the excerpt does not mention powers the government should have but powers it should not have. The necessary and proper clause has been used over time to expand congressional power, not to limit it.

62 A Objective 008

Response A is correct because Jefferson's appeal, like others at the time, shows how commonplace it was to expect others' help in capturing and returning a human being, seen as stolen property at the time, to enslavement. Response B is incorrect because Jefferson describes Sandy as self-motivated and intelligent enough to do carpentry work, shoemaking, and horse jockeying while also planning an escape from his plantation. Response C is incorrect because Jefferson, in writing this appeal to his fellow Virginians, shows that enslaved people such as Sandy were considered valuable enough to offer a monetary reward for their capture and return. Response D is incorrect because Jefferson does not allude to any emancipation of Sandy, nor were any significant number of politically powerful enslavers emancipating enslaved people at this time.

63 A Objective 012

Response A is correct because the colonial press allowed, and at times encouraged and profited from, appeals like Jefferson's, which promoted the belief held by enslavers that enslaved people were nothing more than their property. Response B is incorrect because, although Sandy is described as having many skills beyond agricultural work, the source would not be particularly valuable or robust enough to engage students in meaningful analysis of the topic. Response C is incorrect because there is no mention of a labor shortage in the excerpt beyond the loss of one skilled worker; therefore, the excerpt would not be helpful evidence when discussing economic issues. Response D is incorrect because there is no direct mention of the living conditions of Sandy during his enslavement in this excerpt, nor would Jefferson's appeal likely display a true account of the housing of enslaved people.

64 D Objective 008

Response D is correct because this excerpt shows the diversification of skill sets and quality production of enslaved people in the South. Response A is incorrect because there is no quantity listed for the percentage of enslaved people who were deemed "skilled artisans," who may have had more unique living and working conditions than enslaved people on plantations, based on this excerpt alone. Response B is incorrect because, other than the reference to enslaved people who "have the opportunity to be instructed," there is no mention of any sort of apprenticeship via slaveholders' own investment. Response C is incorrect because there is no mention of an enslaved person's ability to socially climb, be emancipated, or collect payment for their work based on this excerpt.

65 B Objective 012

Response B is correct because the excerpt provides a one sided and brief secondary synopsis of the variety of skills of enslaved people in the South prior to the Civil War. It does not include any reference to opinions or viewpoints of the enslaved people themselves, which would provide a useful contrast when studying this topic in a classroom. Response A is incorrect because, although U.S. census data were first collected in 1790, there was little to no data regarding the names and work skills of enslaved people provided in the census. Response C is incorrect because business ledgers would not likely show a contrast to the points made in the original excerpt. Response D is incorrect because a textbook excerpt on the southern agrarian economy would not provide contrast to the Stampp excerpt.

66 A Objective 008

Response A is correct because the Northwest Ordinance, passed by the Confederation Congress in 1787, fostered development in western lands by providing for surveying and dividing federal lands into an orderly grid. Response B is incorrect because the Louisiana Purchase applied to lands west of Kentucky and the Northwest Territories. Response C is incorrect because the passage does not discuss issues related to Native Peoples. Response D is incorrect because the passage focuses on property rights, not the War of 1812.

67 D Objective 010

Response D is correct because the passage describes the way in which public policy, in this case the Northwest Ordinance of 1787, can influence where people decide to live and work. Response A is incorrect because the passage describes the pull factor of clearer land ownership laws in the Northwest Territories; it does not describe a push factor such as a natural disaster or social unrest. Response B is incorrect because the passage does not describe cultural diffusion or cultural convergence. Response C is incorrect because the passage does not discuss cultural identity.

68 B Objective 008

Response B is correct because the  Mo hawk  were one of six Nations (Oneida, Onondaga, Cayuga, Seneca, and Tuscarora) of the Haudenosaunee Confederacy, excluding the surrounding Algonquin-speaking Wampanoag. Response A is incorrect because, although there has been a revival of Native languages in recent decades, English (and in some cases French) is the predominant language used in meetings of both Wampanoag and  Mo hawk  Peoples. Response C is incorrect because, although political leaders of the Wampanoag and  Mo hawk  are given different titles, sachem and chief respectively, this is not considered a characteristic that differentiates the two significantly. Response D is incorrect because the "five civilized tribes" refers to the Creek, Cherokee, Choctaw, Chickasaw, and Seminole Peoples in the southeastern U.S., whereas the  Mo hawk  Peoples mostly live in the northeastern U.S.

69 D Objective 008

Response D is correct because, although the Cherokee resisted colonialism and suffered loss of territory to Europeans, they also allied with the British in the American Revolution and were one of a few Native tribes to enslave Africans. Response A is incorrect because, although they resisted colonialism and suffered loss of territory to Europeans, the Mashpee Wampanoag did not fight on the side of the British in the American Revolution, nor did they enslave Africans. Response B is incorrect because, although the Pueblo resisted Spanish colonialism and suffered loss of territory in the process, they did not side with the British in the American Revolution, nor did they enslave Africans. Response C is incorrect because, although the Shawnee resisted colonialism and suffered loss of territory to Europeans, only some of them allied with the British and they did not enslave Africans.

70 B Objective 008

Response B is correct because the House of Burgesses, part of the bi-cameral legislature of the General Assembly in the colony of Virginia, was modeled after the bi-cameral British Parliament, both democratically representing and enacting laws for its respective citizens. Response A is incorrect because colonial governors were appointed by the British Crown and did not represent the American colonists, unlike British Parliament, which represents the people—not the Crown—via locally elected officials. Response C is incorrect because the colonial courts were established by the British Crown, and judges were appointed, not elected as representatives of local citizens. Response D is incorrect because, although a seat in the House of Burgesses was filled by a member of the College, William and Mary was unlike the British Parliament in that it required a religious component and provided higher education, and was not a lawmaking body.

71 D Objective 008

Response D is correct because weekly newspapers such as The Boston Gazette and pamphlets like "Letters of a Farmer in Pennsylvania" provided each colony with news of each other's efforts and thwarted British efforts to limit colonial unity. Response A is incorrect because the goal of the colonial press was not to undermine religious leaders, who in many cases supported colonial efforts to rebel. Response B is incorrect because, although the press pushed people to side with Loyalists or Patriots, the media in the eighteenth century colonies did not directly create political parties. Response C is incorrect because, although the press published images of violent acts against colonists by the British (Paul Revere's "The Boston Massacre"), there was no direct causal relationship between what was printed and violent action against British colonial authorities or Loyalists.

72 C Objective 008

Response C is correct because the predominant debate between northern and southern states regarding slavery was exacerbated by the Louisiana Purchase, forcing a determination of whether slavery would be legal or not in the newly acquired lands. Response A is incorrect because, although Native Peoples were ultimately pushed off much of their ancestral lands due to increased numbers of American settlers pushing westward, this was not a major point of contention among regions of the United States. Response B is incorrect because although there was suspicion from New England merchants and factory owners regarding economic losses from newly opened ports and lands in the west, this did not significantly contribute to increased sectionalism in the country. Response D is incorrect because most American citizens were free to access and settle in lands acquired via the Louisiana Purchase, and this access did not particularly contribute to increased sectionalism in the country.

73 C Objective 009

Response C is correct because the explosion onboard the U.S.S. Maine in 1898 was wildly covered in the so-called yellow press. Sayings like "Remember the Maine" were popularized by the yellow press and even, in the case of this excerpt, in songs, which the United States used to justify war against Spain in Cuba, Puerto Rico, and the Philippines. Response A is incorrect because the excerpt does not mention guerilla tactics, and the cause of the explosion on board the U.S.S. Maine was likely a faulty boiler, not sabotage. Response B is incorrect because the excerpt contains no references to headlines by the yellow press. Response D is incorrect because muckraking journalists focused their attention on domestic concerns like issues in the American food industry, while reporting on the Spanish–American War was jingoistic and not often investigative in nature.

74 A Objective 012

Response A is correct because the excerpt provides evidence of the extent to which popular media and culture can affect and be affected by U.S. foreign policy. Response B is incorrect because the excerpt does not provide evidence of effective military strategies. Response C is incorrect because the excerpt does not refer to the Bay of Pigs invasion, which occurred over half a century after the explosion onboard the U.S.S. Maine. Response D is incorrect because the excerpt in and of itself cannot be used to examine a larger issue of historical narratives.

75 C Objective 009

Response C is correct because the goal and result of the Indian Removal Act of 1830 was to offer financial incentives for Native Peoples in the Southeast to relocate west of the Mississippi River. Over fifty thousand Native Peoples were forced off their land, resulting in the forced migration known as the Trail of Tears, in which over six thousand Cherokee died on the march to Oklahoma. Response A is incorrect because the Dawes Act of 1887, which authorized the federal government to divide reservations into individual plots of land for sale, occurred almost sixty years after the Indian Removal Act. Response B is incorrect because the displacement of Native Peoples in the Ohio River Valley was not a result of the Indian Removal Act of 1830 as the act did not apply to the Northwest Territories. Response D is incorrect because the forced placement of Native children into boarding schools occurred in the mid-nineteenth century.

76 B Objective 009

Response B is correct because the Democratic Party, by 1856 an essentially anti-abolitionist party, wanted to expand slavery into newly formed states in the West, effectively forcing "free soilers" to adopt enslavement practices in the eyes of Republicans. This is represented in the cartoon by the four smaller men holding down the giant man, a "free soiler," and forcing an enslaved man down his throat. Response A is incorrect because the cartoon shows violence being perpetrated against abolitionists by proslavery forces. Response C is incorrect because the Republican Party in 1856 was anti-slavery and would not be supported by the proslavery forces. Response D is incorrect because the Democratic Party was promoting popular sovereignty in the newly formed western states in order to expand, not stop, slavery. This is shown in the cartoon by the Democratic Party platform with the words "Kansas," "Cuba," and "Central America" illustrating the desire of the Democratic Party to spread slavery westward and even outside the borders of the United States.

77 A Objective 009

Response A is correct because J.B. Elliot's 1861 political cartoon of a snake slowly squeezing its prey depicts Union General Winfield Scott's plan, often referred to as the "anaconda plan," to launch a naval offensive to blockade Confederate troops along the two coasts and down the Mississippi River. Response B is incorrect because the cartoon's depiction of Union forces as a snake does not present a Confederate perspective nor a predicted Confederate victory. Response C is incorrect because, although this map shows economic sectors that could be disrupted by Scott's offensive directed at major ports, the exchange of raw cotton and enslaved people were the major targets of the offensive. Response D is incorrect because there is no detail of western territories included in Elliot's map, nor is there indication of where specifically the Confederacy would attempt to defend itself.

78 D Objective 009

Response D is correct because, although the Fifteenth Amendment was ratified in 1870, extending the right to vote to Black men, by the  18 nineties  many Southern states had enacted legislation to effectively disenfranchise these new voters until the Voting Rights Act of 1965 provided federal voting protections. Response A is incorrect because, from a federal standpoint, there was no legislation specifically regarding workplace discrimination developed in the Reconstruction era. Response B is incorrect because the Great Migration of African American families in the early twentieth century brought millions of people to northern cities to create new communities and freely share ideas. Response C is incorrect because, although securing property rights protections for African Americans was a challenge during both eras, it was not more relevant to proponents of the Civil Rights Movement than securing the right to vote.

79 B Objective 009

Response B is correct because in 1925 John Scopes was convicted and fined  100 dollars  for teaching evolution in Dayton, Tennessee, inciting a highly publicized debate over religious traditionalism and modernity in the United States. Response A is incorrect because, although there were significant cultural and societal changes because of the Great Migration, it was not a symbol of the conflict between traditionalism and modernity in the United States. Response C is incorrect because the influenza pandemic of 1918 was a medical catastrophe that was not a symbol of the cultural conflict between traditionalists and modernists at the time. Response D is incorrect because, although the cultural flourishing that emanated from the African American community within Harlem challenged some traditional ideas of art at the time, the Harlem Renaissance was not a symbol of conflict among the modernists and traditionalists in a larger sense.

80 A Objective 009

Response A is correct because the Stonewall Rebellion, where members of the LGBTQ community in New York City resisted arrest in 1969, resulted in the launching of the Gay Liberation Front, later the Gay Activists Alliance, and more broadly the LGBTQ rights and Gay Pride movement in the United States. Response B is incorrect because the Wounded Knee incident, considered by many historians to be a massacre, left nearly 300 Lakota People dead at the hands of the U.S. Army in 1890 and was not related to the LGBTQ rights movement. Response C is incorrect because, although the March on Washington was set in a backdrop of civil rights for all, this march in 1963 aimed primarily to advocate for Black Americans, not specifically the LGBTQ community. Response D is incorrect because, although the Summer of Love in 1967 ran on a platform of free love, this anti-war and mostly anti-consumerism movement was not specifically geared toward promoting LGBTQ rights.

81 D Objective 009

Response D is correct because Panama won independence from Colombia in 1903 with the support of the United States, who gained land rights to build a canal in return for protecting Panamanian independence. Response A is incorrect because, although a 10-mile-wide stretch of land along the canal construction zone was acquired by the United States, it was not Roosevelt's intention to colonize the newly independent state of Panama. Response B is incorrect because, although the U.S. construction effort replaced an earlier French canal venture, Roosevelt saw the prospects for a canal as having value in its own right beyond deterring European activities. Response C is incorrect because, although supporting the Panamanian revolution weakened the influence of Colombia in the region, Roosevelt was not primarily motivated by concerns about the power of South American nations.

82 A Objective 010

Response A is correct because in the three examples given in the question, Sparta, Persia, and Carthage represent polities that exercised longstanding hegemonic power, while Athens, the Greek city-states, and Rome represent the new, aspiring powers that rose to challenge the dominance of their powerful neighbors. Response B is incorrect because, while some of the pairs of powers worshipped different faiths, the conflicts between these places and peoples were not primarily based on religion. Response C is incorrect because in these examples there was no third superpower nearby with whom the Athenians, Greeks, and Romans aligned. Response D is incorrect because fertile frontier lands were not present in these examples, specifically in the case of Rome and Carthage, which were separated by the Mediterranean Sea.

83 C Objective 010

Response C is correct because Lake Victoria is considered to have been created by the tectonic shifting of the Great Rift Valley and is located in the area between the two main branches of the valley. Response A is incorrect because the Arabian Peninsula is in southwestern Asia, whereas Lake Victoria is in East Africa, with waters in Tanzania, Uganda, and Kenya. Response B is incorrect because the Indo-Gangetic Plain is located in the northern part of the Indian subcontinent of Asia. Response D is incorrect because Cape Horn is located at the southern tip of South America, whereas Lake Victoria is in East Africa.

84 A Objective 010

Response A is correct because Thailand is located in the region that lies in Southeast Asia, between the eastern border of India, China's southern border, and between the Indian and Pacific Oceans. Response B is incorrect because Korea is far north of Southeast Asia, being located northeast of China and in the region of East Asia. Response C is incorrect because Bangladesh, though very close to Southeast Asia, is in Central Asia and is surrounded on most sides by India. Response D is incorrect because Nepal is considered part of Central Asia, located north of India.

85 A Objective 010

Response A is correct because the large interior of Australia is made up of desert and grasslands, pushing most agriculture and settlement to coastal regions and the more temperate southeastern region. Response B is incorrect because, although the Darling Range is in southwestern Australia, it is not a particularly large range and does not greatly impact overall settlement and agriculture of the country. Response C is incorrect because the monsoons that reach Australia impact the northern half of the continent far more than the southern half. Response D is incorrect because, due to lack of volcanic activity and glacier formations millions of years ago, Australia overall has less fertile soil than many other parts of the world, with no significant fertility differential in the west.

86 B Objective 010

Response B is correct because Mongol conquerors beginning with Genghis Khan extended their rule over an area from Korea to the Balkan peninsula, inaugurating a period known as Pax Mongolica. During this time, political unification allowed cultures, commerce, and technologies to diffuse across this immense area. Response A is incorrect because historians recognize a period of Enlightenment in Eurasia beginning in the seventeenth century, not the thirteenth and fourteenth centuries. Response C is incorrect because the primary outbreak of the Bubonic plague in the mid-fourteenth century occurred too late to be a plausible cause of the period of stability and cross-cultural cooperation. Response D is incorrect because Buddhism spread through many of the areas of Asia hundreds of years before the thirteenth century. Several Mongol Khanates adopted the predominant Islamic faith of the areas they had conquered in southwest Asia, but this postdated Mongol expansion rather than being a condition that made Pax Mongolica possible.

87 C Objective 010

Response C is correct because the process of Sinicization, as described in the passage, demonstrates features of both cultural diffusion and cultural convergence. Diffusion refers to the outward spread of a culture to other peoples, as when Chinese cultural practices gradually expanded outward from the heartland of the Huang He (Yellow) and Chang Jiang (Yangtze) Rivers. Convergence occurs when different cultures begin to blend or share cultural features, as when the Mongol conquerors of the Song Dynasty adopted many of the cultural practices of the conquered population. Response A is incorrect because the passage does not discuss power differentials between social classes. Response B is incorrect because the discussion of Sinicization does not revolve around the interaction of geographic and historical factors. Response D is incorrect because the passage refers to the conquest of peoples without discussing ethnocentrism and oppression.

88 A Objective 010

Response A is correct because the climate of Mesopotamia was semi-arid, making highly productive agriculture possible only in the presence of irrigation technology. Food surpluses resulting from irrigation made a complex urbanized society possible. Response B is incorrect because, although terraced agriculture was known in some parts of the Fertile Crescent, this practice is more associated with other parts of Asia and was not the crucial technology that made complex society possible in ancient Mesopotamia. Response C is incorrect because ancient Mesopotamia was home to civilizations in the Bronze Age and Iron Age, but metals technologies were more a result than a cause of the construction of complex urbanized societies. Response D is incorrect because ancient Mesopotamians did not engage in the large-scale diversion of rivers but the construction of irrigation canals originating from the Tigris and Euphrates Rivers.

89 D Objective 011

Response D is correct because the federal government was not able to tax American citizens under the Articles of Confederation; instead, states were left in charge of raising revenue through taxes, which left the national treasury nearly bankrupt. Response A is incorrect because, although the national government under the Articles could not regulate foreign commerce, they did have the power to negotiate treaties with foreign nations. Response B is incorrect because the national government under the Articles of Confederation had the power to declare war and appoint military officers. Response C is incorrect because the national government was able to acquire and admit new territories under the Articles, as shown by the enactment of the Northwest Ordinance in 1787.

90 C Objective 011

Response C is correct because U.S. residents pay both state (income, sales, excise) and federal (income, gift, estate) taxes. Article II, Section 8 of the U.S. Constitution gives Congress the power to tax; however, states still retain the power to tax residents directly. Response A is incorrect because only the federal government has the power to print money. Response B is incorrect because only states retain the power to issue driver's licenses to their residents as it is not a power given to the federal government in the Constitution. Response D is incorrect because only the federal government has the power to enforce treaties with foreign powers.

91 C Objective 011

Response C is correct because serving on a jury is considered a right as it helps to preserve constitutional freedoms, and it is also a legal obligation for citizens ages 18 and above who do not claim an exemption. Response A is incorrect because citizens are not legally obligated to vote in any election, though it is considered a right to do so. Response B is incorrect because paying state and federal taxes is not considered a right. Response D is incorrect because it is the right of any person accused of a crime to request legal representation, but it is not a defendant's obligation to seek counsel.

92 D Objective 011

Response D is correct because people and interest groups have the First Amendment right to "petition," or lobby, their government representatives by email, phone, or in person to express policy preferences. Response A is incorrect because the establishment clause prohibits Congress from creating or preferencing any church but does not protect lobbying efforts of interest groups. Response B is incorrect because the freedom of the press applies to news organizations sharing information with the public but not to lobbying efforts of interest groups. Response C is incorrect because the right to peaceably assemble allows for people to gather for public protests but does not protect the lobbying efforts of interest groups.

93 B Objective 011

Response B is correct because taking informed action requires that students must first do research to become more informed of policies or current topics, and then engage in democratic society outside of the classroom through some form of action such as a letter-writing campaign to the president. Response A is incorrect because, although to model calling a Senator would be helpful for students to see how easy it is to contact their representative, this does not show students how to gather information before calling their representative, which is an important aspect of taking informed action. Response C is incorrect because, although seeing where representatives meet and work could be helpful for students, just visiting a building does not get students involved or allow them to take any civic action. Response D is incorrect because a meaningful debate scenario, even of current public policy topics and with ample research beforehand, does not result in students engaging with their government representatives or society at large.

94 C Objective 011

Response C is correct because Brown  versus  Board of Education ruled against the decision of Plessy  versus  Ferguson by claiming that racial segregation was unconstitutional, paving the way for desegregation and legally backed equal rights for African Americans. Response A is incorrect because Dred Scott made the claim that African Americans could not be U.S. citizens and therefore they were not protected by Constitutional rights. Response B is incorrect because the Court's decision in Plessy was that racial segregation was constitutional, thus violating the equal rights of African Americans. Response D is incorrect because District of Columbia  versus  Heller, not a case rooted in minority rights, found that citizens had a Constitutional right to bear arms per the Second Amendment.

95 B Objective 011

Response B is correct because the four pillars in this cartoon all refer to a "FREE PRESS," without which the ceiling of "DEMOCRACY" would topple. Response A is incorrect because there is no explicit reference to the First Amendment, nor is a symbol of the First Amendment acting as a shield to the four pillars of "FREE SPEECH" in this cartoon. Response C is incorrect because, although op-ed columns may support or dismiss political candidates, the ceiling of the building in this cartoon is not representing candidates but the idea of a functioning democracy. Response D is incorrect because, although this may be true, there is no indication that the pillars representing a general "FREE PRESS" are in competition with each other; in fact, they seem to be working together to hold the ceiling of "DEMOCRACY" up.

96 A Objective 011

Response A is correct because media consolidation is the process by which media ownership is increasingly concentrated in the hands of fewer people over time. This has led to more focus on coverage of national politics and government at the expense of local political coverage. Response B is incorrect because media consolidation has more likely decreased editorial diversity. Response C is incorrect because media consolidation has not resulted in a wider variety of opinions on the news. Response D is incorrect because people increasingly consume more of their news on social media platforms.

97 C Objective 012

Response C is correct because an activity that involves appropriating native dress and culture in a reenactment of the Thanksgiving feast often perpetuates a problematic narrative wherein Native Peoples are presented as a monolithic culture in the past. Response A is incorrect because Thanksgiving is an appropriate time for an educator to discuss and celebrate the diversity of cultural foods served at various Thanksgiving meals. Response B is incorrect because Thanksgiving is an appropriate time for students to read primary sources from the perspectives of both colonists and Native Peoples. Response D is incorrect because Thanksgiving is an appropriate time for educators to discuss the various ways in which cultures express gratitude with students.

98 C Objective 012

Response C is correct because Jackson's support of the Indian Removal Act and refusal to enforce the Supreme Court's ruling in Worcester  versus  Georgia led to the forced removal of thousands of Cherokee People from their ancestral lands in the event known as the Trail of Tears. Response A is incorrect because there have been no new revelations about Jackson's abuse of power while in office. Response B is incorrect because Jackson's paying off federal debt has been an uncontroversial achievement and has been overshadowed by his more consequential vetoing of the recharter of the Second Bank of the United States. Response D is incorrect because Jackson's support of the spoils system has been seen in more negative light by many historians.

99 B Objective 012

Response B is correct because displaying a holiday tree could be seen as preferring the religious traditions of Christianity over others. Having students participate in decorating the tree could alienate students who are not of the dominant culture or religion of the class and the broader community. Response A is incorrect because discussing the diversity of cultural traditions of gift giving and candle lighting is an inclusive activity appropriate for a lesson about winter holidays. Response C is incorrect because studying the diversity of religious practices is an inclusive activity appropriate for a lesson about winter holidays. Response D is incorrect because having students read primary sources that represent diverse viewpoints is an inclusive activity appropriate for a lesson about winter holidays.

100 B Objective 012

Response B is correct because Kipling's "The White Man's Burden" presents a paternalistic and stereotypical view of the people who were victimized by imperialism and therefore requires proper contextualization by a teacher to be of educational value. Response A is incorrect because, while some content may require context, the information in a journal would likely not be as problematic as "The White Man's Burden." Response C is incorrect because a visual depiction of the Partition of Africa presents no bias and is helpful for understanding imperialism. Response D is incorrect because an encyclopedia article about the African railway system is less likely to contain bias than Kipling's poem.

Total Correct: Review your results against the test objectives.

Open Responses, Sample Responses, and Analyses

Question Number Your Response
Read about how your responses are scored and how to evaluate your practice responses
101

Open Response Item

You may type your written response on the assigned topic in the box provided.


First Sample Weak Response

First Sample Weak Response to Open-Response Item Assignment #1

A theme in the excerpt from The Piano Lesson by August Wilson is resistance to help and being stuck in your ways. Avery is trying to help Berniece, but she is stuck in her ways and refuses his help. Since her mother died, Berniece doesn't play the piano anymore. Her mother used to make her play it. She told her that she could hear her daddy's voice when she played. Berniece became afraid that playing the piano would wake the spirits. That is not a rational thought. Avery tries to help Berniece by getting her to abandon her irrational way of thinking. "You got to put all that behind you, Berniece."

August Wilson uses a quotation to develop the theme of resistance to help and being stuck in your ways. He quotes the Bible when he says, "The Bible say 'Make a joyful noise unto the Lord.'" Berniece is not interested in learning about the Bible or trying anything new. Avery invites her to start a choir at the church, but she doesn't want to. She is just very stubborn, and even at the end of the play Avery tries to talk her into playing the piano and she refuses, "I done told you I don't want to play that piano. Now or no other time." This supports the idea of being stuck in your ways. The excerpt shows how unless you accept help from someone and try something new you are just going to keep doing the same thing over and over. You will be stuck in a rut.

First Weak Response Analysis

Analysis of First Weak Response to Open-Response Item Assignment #1

This is an example of a weak response because it is characterized by the following:

Purpose: The purpose of the assignment is partially achieved. While it is not entirely untrue that Berniece is portrayed as resistant to help and stuck in her ways, these are not significant themes in the passage; they are merely character traits. We do not know that Berniece is resistant to help; we only know that she resists Avery's help on this occasion. Regarding the literary device, the response provides no explanation as to how Wilson uses the quotation, specifically, to develop the theme.

Subject Matter Knowledge: There is a limited application of subject matter knowledge. The proposed themes are not properly stated as themes ("Resistance to help inhibits personal growth" would be better), and no argument is provided to show that the quotation is being used as a literary device. The response also makes some inaccurate claims about the text, such as the claim that Berniece is not interested in learning about the Bible.

Support: The supporting evidence is limited. Although the response provides some support for its conclusions, it also neglects evidence that, if considered, would lead to more plausible conclusions. The response fails to consider Berniece's strong attraction to the piano, which is suggested both by the stage direction at the end and by Avery's first line, which suggests that Berniece is trying to protect the piano from Boy Willie.

Rationale: The response reflects a limited, poorly reasoned understanding of the topic. Although most of the claims in this response are not entirely untrue, the response fails to grasp the larger themes of the passage. The response reduces the conflict to a simple conflict between Avery and Berniece, when the bigger conflict takes place between Berniece and herself. Additionally, much of the first paragraph consists of plot summary.

Second Sample Weak Response

Second Sample Weak Response to Open-Response Item Assignment #1

This excerpt is about how parents sometimes push their kids to perform and excel in things the kids don't want to do, and it ends up backfiring. Berniece's mother forced Berniece to take piano lessons even though she didn't want to. She said she could hear her dead husband, Berniece's daddy, talking when she played it, so Berniece felt pressured to play it. She was the last vestige of her mother's connection to her father, who now was dead. Berniece's mother leaned on Berniece to satisfy her emotional needs, which is the opposite of what a mother should do. Berniece should not have had to take care of her mother. In result she came to resent her mother and the piano and it became like a giant millstone around her neck to the point that now she doesn't ever want to play it. The author uses dramatic irony in this scene because Berniece is making her own daughter do exactly the same thing that her mother made her do by playing on the piano. "I got Maretha playing on it. She don't know nothing about it. Let her go on and be a schoolteacher or something. She don't have to carry all of that with her. She got a chance I didn't have. I ain't gonna burden her with that piano." Berniece has her own vision for what Maretha ought to be rather than just supporting Maretha in whatever decision she makes. Otherwise, she is going to end up resenting Berniece just like Berniece resents her own mother.

Second Weak Response Analysis

Analysis of Second Weak Response to Open-Response Item Assignment #1

This is an example of a weak response because it is characterized by the following:

Purpose: The purpose of the assignment is partially achieved. A theme and a literary device are identified, but neither is appropriate for the excerpt. The response focuses on what is essentially a backstory—the relationship between Berniece and her mother—and ignores the immediate conflict between Berniece and Avery. Avery is also trying to "push" Berniece "to perform," but the response inexplicably fails to consider this.

Subject Matter Knowledge: There is a limited application of subject matter knowledge. Irony is not an appropriate literary device for the excerpt. It might be ironic if Berniece were actually forcing Maretha to do the very thing her own mother forced her to do (in which case it would be an example of situational irony, not dramatic irony). However, that is not the case: Berniece tells Avery that she is determined not to do to Maretha what her own mother did to her.

Support: The supporting evidence is limited. The sole quotation is too long, and most of the quotation is either irrelevant to or contradicts the point the response wants to make. The quotation includes the sentence "She don't know nothing about it," but the response never considers what "it" refers to.

Rationale: The response reflects a limited, poorly reasoned understanding of the topic. The response reduces the excerpt to the backstory of the relationship between Berniece and her mother, in the process losing other important elements of the excerpt: the dispute with Boy Willie, Berniece's desire to keep (and perhaps play) the piano, and Avery's diagnosis of Berniece and his attempts to help.

First Sample Strong Response

First Sample Strong Response to Open-Response Item Assignment #1

A theme in this excerpt is that people sometimes fail to realize their potential due to their own fears or inhibitions. Berniece is apparently a gifted pianist but has stifled her talent due to some trauma in her past and her fear that playing the piano will cause that trauma to resurface. To protect herself, Berniece has literally "shut the top" on the piano, and, figuratively, on her potential.

Berniece endows the piano with a terrifying magical power. From her mother she learned both the ability to play and the belief that her playing had the power to "wake them spirits" of the dead. Berniece is afraid of this magical power—the source of her reputed power as a pianist—and wants no part of it. Berniece instead teaches her daughter to play the piano, but her goals for Maretha are more practical than magical. She wants her to become "a schoolteacher or something."

Avery sees that Berniece is allowing her fears to stifle her talent and urges her "to put all that behind you." Using a metaphor, Avery compares Berniece's fears to a sack of stones. The metaphor suggests that Berniece has a choice—either to carry the sack of stones and be perpetually burdened by it or to "set it down and walk away from it." Avery speaks in repetitive phrases ("You can walk over there right now"), almost as if to hypnotize Berniece into overcoming her fears.

Although Berniece repeatedly says that she doesn't want to play the piano, Avery seems to think this is just a defense mechanism. A stage direction has Berniece moving towards the piano, suggesting that she is strongly drawn to it. This is a climactic moment. Just as she seems about to play the piano, she collects herself and reaffirms her refusal. She is not yet able to confront her fears.

First Strong Response Analysis

Analysis of First Strong Response to Open-Response Item Assignment #1

This is an example of a strong response because it is characterized by the following:

Purpose: The purpose of the assignment is fully achieved. A significant theme is identified in the first paragraph and discussed throughout the response. Specific literary devices are identified in the first paragraph, which implies that the piano may be read as a symbol for Berniece's creative potential, and in the third paragraph, which demonstrates that Avery uses metaphor and repetition to help Berniece overcome her fears.

Subject Matter Knowledge: There is a substantial application of subject matter knowledge. The response accurately identifies and provides examples of metaphor and repetition, distinguishes between literal and figurative uses of the piano, and appropriately deploys discipline-specific terminology like stage direction and climactic moment.

Support: The supporting evidence is sound. Quotations are shrewdly selected, succinct, and always in service of the point being made. When the response claims that Berniece's goals for Maretha are practical in nature, the reason for that claim ("a schoolteacher") is provided; when it claims that Avery speaks in repetitive phrases, an example of those phrases is provided.

Rationale: The response reflects an ably reasoned, comprehensive understanding of the topic. A number of subtle inferences are drawn from the excerpt and carefully identified as such: Berniece is "apparently" a gifted pianist; Avery "seems to think" that Berniece's protestations are just a defense mechanism. The response effectively labels and categorizes parts of the excerpt, such as when it claims that Berniece is "afraid" of her power and when it claims that Berniece's goals for Maretha are "more practical than magical."

Second Sample Strong Response

Second Sample Strong Response to Open-Response Item Assignment #1

This excerpt dramatizes the theme of generational trauma—the ways that parents pass their psychological burdens to their children. In Wilson's play, the piano becomes the symbol of a trauma transmitted through three generations, from Berniece's mother, to Berniece, and, unless something is done about it, to Berniece's daughter.

Berniece's mother's suffering, seemingly triggered by her husband's death, could be due more generally to a loss of her past or her family history. It manifests in bizarre behavior such as talking to pictures of dead loved ones. Even more bizarrely, Berniece's mother forces Berniece to play the piano, telling her that her playing channels the voice of her dead father. That's a lot of psychological baggage to lay on a child. Berniece projects her ambivalent feelings about her mother—a tangle of devotion, shame, and resentment—onto the piano. For Berniece, the piano symbolizes her feelings about her mother. For the audience, it symbolizes the burden of inherited trauma.

Berniece is determined to avoid her mother's fate: "I said that wasn't gonna happen to me," she says. By refusing to play the piano, she is silencing her father's voice and her family history. She is determined not to transmit that painful past to her daughter, Maretha. "She don't know nothing about it," Berniece says. She believes that by refusing to play the piano she is choking off that past. In this way she will end the cycle of generational trauma, and Maretha won't "have to carry all of that with her."

To Avery, Berniece is simply repressing the trauma, not ending it. By repressing it, Berniece is in danger of unwittingly transmitting it to Maretha. Avery encourages Berniece to break the cycle of generational trauma by confronting the piano that symbolizes it. Only by playing the piano again can she "put all that behind" her.

Second Strong Response Analysis

Analysis of Second Strong Response to Open-Response Item Assignment #1

This is an example of a strong response because it is characterized by the following:

Purpose: The purpose of the assignment is fully achieved. A significant theme is identified in the first paragraph and discussed throughout the response. A specific literary device is identified in the first paragraph. The remainder of the response is consistently devoted to an explanation of how the piano functions as a symbol of generational trauma and of Berniece's conflicted feelings about her mother.

Subject Matter Knowledge: There is a substantial application of subject matter knowledge. The discussion of how the piano functions as a symbol both for the characters in the play and for the play's audience is a careful and nuanced handling of the literary device. The application of psychological concepts such as trauma, ambivalence, and repression focuses the analysis and demonstrates knowledge of a cultural context relevant to the excerpt.

Support: The supporting evidence is sound. Although only two characters appear in the excerpt, the response uses textual evidence to elaborate a reading that illuminates the relationships between four characters—Berniece, Berniece's mother, Maretha, and Avery. Using the available evidence, the response draws a complex portrait of Berniece's mother that effectively turns her into a central character in the drama. Quotations are succinct and relevant to the chosen theme.

Rationale: The response reflects an ably reasoned, comprehensive understanding of the topic. It illuminates in turn the motives of the several characters—first Berniece's mother, then Berniece, then Avery—all while maintaining its focus on the theme of generational trauma and the symbol of the piano.

102

Open Response Item

You may type your written response on the assigned topic in the box provided.


First Sample Weak Response

First Sample Weak Response to Open-Response Item Assignment #2

The differing philosophies of the political parties meant that Thomas Jefferson had to constantly argue for his own philosophy during his presidency. It put him on the defensive. In Source 1, Thomas Jefferson is shown as being opposed to the party of George Washington and John Adams. He is shown with a bottle of Brandy being helped by the Devil, who he calls "Old Friend." The purpose of this source is to attack Jefferson because of his political philosophy. He was seen as having a destructive force on the federal government.

In Source 2 Jefferson speaks for himself, explaining his political philosophy. He asks the people to abide by the Constitution, to follow "the will of the law" and "the common good." He says that "Every difference of opinion is not a difference of principle," which shows that he was able to find common ground with Washington and Adams. His purpose was to argue against those who felt that 'a republican government cannot be strong,'" Jefferson showed that it could be strong by enacting the Louisiana Purchase. This was a key development in Jefferson's presidency. This was a large tract of land purchased from France that doubled the national territory. His purpose in the Second Inaugural Address is to argue against those who felt the Louisiana Purchase "would endanger its union." Jefferson replied that "who can limit the extent to which the federative principle may operate effectively?" Jefferson believed that by enlarging the territory would only make the nation stronger because it would be settled by fellow Americans rather than "by strangers."

First Weak Response Analysis

Analysis of First Weak Response to Open-Response Item Assignment #2

This is an example of a weak response because it is characterized by the following:

Purpose: The purpose of the assignment is partially achieved. The claim in the first sentence, while not untrue, is so general that it goes without saying. The response gives no accurate description of Jefferson's philosophy or that of his opponents. Analysis of the sources is also partial. Jefferson's purpose in the first inaugural address is not, as the response suggests, to explain his political philosophy; while it is not inaccurate to say that his purpose was "to argue against those who felt that 'a republican government cannot be strong,'" we still do not know who those people are or what Jefferson would argue in rebuttal.

Subject Matter Knowledge: There is a limited application of subject matter knowledge. The two parties are not identified nor their philosophies discussed. "The party of George Washington and John Adams" needs qualification, as Washington did not identify, as Adams did, as a Federalist.

Support: The supporting evidence is limited. The purpose of some of the quotations is misrepresented. For example, while Jefferson does indeed ask the people to abide by the Constitution and follow the will of the law, he is simply asking for an orderly transfer of power; he does not intend this as an explanation of his political philosophy.

Rationale: The response reflects a limited, poorly reasoned understanding of the topic. The discussion of the Louisiana Purchase, in particular, is poorly reasoned. The response does not consider why the Federalists would allege that the Louisiana Purchase would endanger the union. Jefferson's defense of the Louisiana Purchase is made entirely through quotation and paraphrase; the response provides little of its own reasoning as to why Jefferson thought it would strengthen the nation.

Second Sample Weak Response

Second Sample Weak Response to Open-Response Item Assignment #2

The philosophies of the political parties had major effects on developments during the Jeffersonian administration. The Federalists in the North adopted a policy of tax and spend, while the Republicans in the South argued for fiscally conservative values. Jefferson was a Republican who wanted to cut Washington pork barrel spending, balance the budget, and cut taxes. Washington and Adams and the Federalists mocked Jefferson as an alcoholic and an atheist (Source 1) because he was a threat to Northern elites. He believed in the value of a free press, he protected the rights of Americans to "think freely, and speak and write what they think" (Source 2). Reason should be allowed to combat "error of opinion." This would happen naturally if the federal government got out of the way and left decision making to the people. Jefferson thought that government should be stripped of "unnecessary offices, useless establishments and expenses." Government was bloated and corrupt, the people should decide how to spend their own hard-earned money. Jefferson was opposed to the national bank, yet another example of governmental overreach. Jefferson wanted to expand the federative principle through the Louisiana Purchase, spending 15 million to buy a tract of land from France that would enlarge the size of the nation so that all may enjoy the fruits of liberty. This led to the Lewis and Clark expedition blazing a trail to the Pacific and opening up the nation.

Second Weak Response Analysis

Analysis of Second Weak Response to Open-Response Item Assignment #2

This is an example of a weak response because it is characterized by the following:

Purpose: The purpose of the assignment is partially achieved. The response constructs a claim that the philosophies of the parties "had major effects" on developments during Jefferson's presidency, but it does not tell us what those effects were. Point of view in the cartoon is misrepresented when the response claims that Washington and Adams mocked Jefferson as an alcoholic and atheist. The central idea and purpose of the two inaugural addresses is misrepresented.

Subject Matter Knowledge: There is a limited application of subject matter knowledge. Although the response makes some accurate historical claims (for example, that Jefferson was opposed to a national bank), it also makes some inaccurate ones. To characterize the Federalists' philosophy as "tax and spend" shows, at best, limited knowledge. The reference to Lewis and Clark, although historically accurate, is not made relevant to the argument.

Support: The supporting evidence is limited. Evidence from the sources is taken out of context and misrepresented. The claim about Jefferson's belief in the value of a free press, while not inaccurate, misses the point Jefferson wants to make in his first inaugural address. It also seems to contradict Jefferson's reprimand of the press in his second inaugural address. Further rationale might have reconciled this apparent contradiction.

Rationale: The response reflects a limited, poorly reasoned understanding of the topic. Despite the response's main claim that the political philosophies affected developments during Jefferson's presidency, those developments are not addressed until the discussion of the Louisiana Purchase at the end of the response. Without further rationale, Jefferson's desire "to expand the federative principle" seems more in keeping with the Federalist philosophy than his own, and the  15 million dollars  expenditure seems inconsistent with Jefferson's allegedly "fiscally conservative values."

First Sample Strong Response

First Sample Strong Response to Open-Response Item Assignment #2

Jefferson ran on a platform of fiscal austerity—cutting taxes, reducing the federal budget, and reducing the size of the military. This reflected the goals of the Democratic-Republican party, which wanted to limit the power of the federal government and give power to the states instead. Federalists, in contrast, stood for a strong federal government, and they saw Jefferson as a threat to their aims. In a Federalist cartoon from the first year of Jefferson's presidency, Jefferson is depicted as "Mad Tom," a drunkard in league with the devil in an effort to pull down the federal government, which, under Washington and Adams, had been built strong like a stone pillar.

Jefferson faced such criticism throughout his two terms. In his first inaugural address, he assured Federalists that he would be a president for all Americans. He argued that, although divided by opinion, Federalists and Democratic-Republicans shared a commitment to a union of republican states. Perhaps too optimistically, he says, "We are all Republicans, we are all Federalists." In particular, he is responding to a "fear that a republican government cannot be strong."

In his second inaugural address, Jefferson lists the accomplishments of his first term (shrinking the bureaucracy, reducing the budget, lowering taxes) and responds to criticism of the Louisiana Purchase. The Louisiana Purchase was an opportunity to expand the nation's size, wealth, and geopolitical power, but Federalists might have seen it as a threat. Jefferson argued that it would only make the nation stronger, as "The larger our association, the less will it be shaken by local passions." Federalists feared that westward settlement would draw power away from urban centers in the Northeast and tip the balance of power toward the Democratic-Republicans. Still, Jefferson counters, isn't it better that the territory is settled "by our own brethren and children, than by strangers of another family?"

First Strong Response Analysis

Analysis of First Strong Response to Open-Response Item Assignment #2

This is an example of a strong response because it is characterized by the following:

Purpose: The purpose of the assignment is fully achieved. The response makes a claim that Jefferson's Democratic-Republican philosophy shaped his program of fiscal austerity and that he was motivated by Federalist concerns about the strength of the federal government. Phrases such as "He argued that" and "He is responding to" announce places where the response is analyzing the central idea, purpose, and point of view of the sources.

Subject Matter Knowledge: There is a substantial application of subject matter knowledge. The two political parties are identified and their philosophies summarized. The claims that "Jefferson ran on a platform of fiscal austerity" and that "Federalists feared that westward settlement would draw power away from urban centers in the Northeast" also show substantial subject matter knowledge.

Support: The supporting evidence is sound. The response uses some combination of summary, paraphrase, and quotation of each of the sources, selecting only the detail necessary for shaping its argument. It shows a keen eye for useful detail, finding in the "stone pillar" a symbol of the Federalist interest in a strong central government.

Rationale: The response reflects an ably reasoned, comprehensive understanding of the topic. Although the response reflects some subject matter knowledge not derivable from the sources, the central claim that "Jefferson faced such criticism throughout his two terms" is grounded in evidence prominent in each of the sources. Also, note how the response tempers claims about which it is not entirely certain: "Perhaps too optimistically," "Federalists might have seen it as a threat." This measured phrasing only makes the claims more plausible.

Second Sample Strong Response

Second Sample Strong Response to Open-Response Item Assignment #2

The Federalist party favored a strong central government, while Jefferson's Republican party thought that authority should be concentrated in the states. When Jefferson was elected president in 1800, he pursued an agenda consistent with Republican philosophy, but continued criticism in the press may have motivated him over time to appease the Federalists.

The cartoon was drawn early in Jefferson's first term, apparently by a Federalist partisan. The central idea is simple: the artist fears that Jefferson will "bring down" the federal government established under Washington and Adams. The purpose is to paint the new president as hostile to orthodox religion and polite society, thus stoking fears in the New England businessmen that constituted the Federalist base.

In his first inaugural address, Jefferson tries to allay these fears by downplaying the differences between the parties, announcing that "we are all Federalists." His repeal of the whiskey tax, however, suggested that he was willing to erode the federal government's tax base for the sake of benefiting agrarian states in the South.

Jefferson was sensitive to criticism. He complains in his second inaugural address that "the artillery of the press has been levelled against us." The criticism may have prompted him to moderate his views. The Louisiana Purchase, for example, seems like it would have appealed more to Federalists than to Republicans. It is not immediately clear how doubling the size of the nation would benefit farmers in Virginia, while it is clear that the cost would have increased their tax liability.

Jefferson's purpose in his second inaugural address is to convince Federalists that enlarging the nation would strengthen the central government: "The larger our association, the less will it be shaken by local passions." This illustrates the extent to which Jefferson modified his philosophy over the course of his presidency.

Second Strong Response Analysis

Analysis of Second Strong Response to Open-Response Item Assignment #2

This is an example of a strong response because it is characterized by the following:

Purpose: The purpose of the assignment is fully achieved. The claim that Jefferson "modified his philosophy" to "appease the Federalists" is made explicitly in the first and last paragraphs. The second paragraph succinctly analyzes, in successive sentences, the point of view, central idea, and purpose of the cartoon.

Subject Matter Knowledge: There is a substantial application of subject matter knowledge. The two political parties are identified and their philosophies summarized. Subject matter knowledge is evident in the claim that the Federalists consisted largely of "New England businessmen" and in the discussion of the whiskey tax and the implications of its repeal.

Support: The supporting evidence is sound. The response provides textual evidence from each of the sources. Jefferson's repeal of the whiskey tax is an apt illustration of the threat that Federalists perceived in his presidency.

Rationale: The response reflects an ably reasoned, comprehensive understanding of the topic. The response uses the press's criticism of Jefferson's presidency to explain what it sees as a shift in Jefferson's political philosophy. The response also argues that the Louisiana Purchase ought to have appealed more to Federalists than to Republicans. These arguments, though not indisputable, demonstrate an ability to deduce plausible conclusions from the available evidence.

Review the Performance Characteristics and Score Scale for Written Performance Assignments.

Multiple Choice Question
Practice Test Evaluation Chart

In the evaluation chart that follows, the multiple-choice questions are arranged in numerical order and by test objective. Check your responses against the correct responses provided to determine how many questions within each objective you answered correctly.

Subarea: Literature and Language

Objective 001: Apply knowledge of literature that represents a range of perspectives reflecting diversity of ability; gender; race; ethnicity; sexual orientation; nation of origin; religion; age; and cultural, economic, and geographic backgrounds.
Question Number Your Response Correct Response
1 B
2 D
3 C
4 C
5 C
6 B
7 B
8 A
9 D

 out of 9

Objective 002: Apply knowledge of informational texts.
Question Number Your Response Correct Response
12 C
13 D
14 B
15 D
16 C
17 A
18 B
19 A

 out of 8

Objective 003: Apply knowledge of theory, research, and instructional practice related to language acquisition and reading.
Question Number Your Response Correct Response
20 C
21 A
22 B
23 B
24 A
25 D
26 A
27 C
28 D
29 A
30 C
31 B

 out of 12

Subarea: Literature and Language (Objectives 001–003) Total  out of 29

Subarea: Expressive and Receptive Communication

Objective 004: Apply knowledge of the writing process and techniques for writing arguments, informative/explanatory texts, and narratives.
Question Number Your Response Correct Response
32 C
33 D
34 D
35 C
36 B
37 A
38 B

 out of 7

Objective 005: Apply knowledge of techniques for conducting academic research.
Question Number Your Response Correct Response
39 C
40 D
41 D
42 A
43 A
44 B

 out of 6

Objective 006: Apply knowledge of techniques for speaking and/or expressive communication and for listening and/or receptive communication in a variety of contexts.
Question Number Your Response Correct Response
45 D
46 B
47 B
48 C
49 D
50 A

 out of 6

Subarea: Expressive and Receptive Communication (Objectives 004–006) Total  out of 19

Subarea: History

Objective 007: Apply knowledge of major social, cultural, political, economic, and technological developments in ancient to early modern societies (to approximately the mid-nineteenth century  C E ) around the world.
Question Number Your Response Correct Response
51 B
53 B
54 D
55 A
56 C
57 D
58 C
59 D

 out of 8

Objective 008: Apply knowledge of major political, economic, social, and cultural developments in the history, prior to 1825, of what is now the United States.
Question Number Your Response Correct Response
60 D
62 A
64 D
66 A
68 B
69 D
70 B
71 D
72 C

 out of 9

Objective 009: Apply knowledge of major political, economic, social, and cultural developments in the history of the United States from 1825 to the present.
Question Number Your Response Correct Response
73 C
75 C
76 B
77 A
78 D
79 B
80 A
81 D

 out of 8

Subarea: History (Objectives 007–009) Total  out of 25

Subarea: Geography, Government, and Civic Life

Objective 010: Apply knowledge of physical and human geography across the following regions: Western Asia, the Middle East, North Africa, Sub-Saharan Africa, Europe, North America, Central America, Caribbean Islands, South America, Central and South Asia, East Asia, Southeast Asia, and Oceania.
Question Number Your Response Correct Response
67 D
82 A
83 C
84 A
85 A
86 B
87 C
88 A

 out of 8

Objective 011: Apply knowledge of government and civic life.
Question Number Your Response Correct Response
61 B
89 D
90 C
91 C
92 D
93 B
94 C
95 B
96 A

 out of 9

Objective 012: Apply knowledge of the guiding principles and instructional practices of effective history and social science education.
Question Number Your Response Correct Response
52 B
63 A
65 B
74 A
97 C
98 C
99 B
100 B

 out of 8

Subarea: Geography, Government, and Civic Life (Objectives 010–012) Total  out of 25

Practice Test Score Calculation

Available Spring 2024.